5/4 hour = __ minutes

Answers

Answer 1

Answer:

hour= 1.25

MINUTES ANSWER= 75 minutes

Step-by-step explanation:

hope that helps>3

Answer 2

Answer:

5/4 hour= 75 minutes

--------------------------------

[tex]\textbf{HOPE IT HELPS}[/tex]

[tex]\textbf{HAVE A GREAT DAY!!}[/tex]


Related Questions

Tim did a survey at a large shopping centre. He asked 400 visitors to the centre to choose the main reason for their visit. The reasons were shops, free parking, food court and location.
From the people taking part in the survey, one person is chosen at random to get a prize. Find the probability a female who chose location gets the prize.

shops free park. food location total
male 88 24 40 21 173
female 110 38 56 23 227
total 198 62 96 44 400​

Answers

Answer:

23 / 400 = 0.0575

Step-by-step explanation:

Given the distribution :

_____shops__ free park. food _: location total

male 88 24 40 21 173

female 110 38 56 23 227

total 198 62 96 44 400​

From the table of the distribution Given, we obtain :

n(Female n location) = 23

n(Total) = 400

Hence, the probability that a female who chose location gets the prize will be ;

n(Female n location) / n(Total) = 23 / 400 = 0.0575

5/6 ÷ 1/3 - 2/3 (2/5)

Answers

Answer:

[tex] \frac{67}{30} \: \text{or} \:2 \frac{7}{30} [/tex]

Step-by-step explanation:

5/6 ÷ 1/3 - 2/3 (2/5)

= 5/6 ÷ 1/3 - 2/3 × 2/5= 5/2 - 2/3 × 2/5= 5/2 - 4/15= 67/30 or 2 7/30

Hope it helps you! \(^ᴥ^)/

For any number n>1, is
|(.5 +.2i)^n|
A. greater than 1?
B. less than 1?
C. equal to 1?
PLZ HELP

Answers

Answer:

B. Less than 1

Step-by-step explanation:

You could plug in values of n greater than 1 and see what happens....

Example n=2 gives |(.5+.2i)^2|

Simplifying inside gives |(.5)^2+2(.5)(.2i)+(.2i)^2|

=|.25+.2i+.04i^2|=|.25+.2i-.04|=|.21+.2i|.

Applying the absolute value part gives sqrt(.21^2+.2^2)=sqrt(.0441+.04)=sqrt(.0841)=.29

This value is less than 1.

We should also be able to do the absolute value first then the power.

|.5+.2i|=sqrt(.25+.04)=sqrt(.29)

So |.5+.2i|^2=.29 which is what we got long way around.

Anyways (sqrt(.29))^n where n is greater than 1 will result in a number greater than 0 but less than 1.

.4.1 Here are the data from Exercise 2.3.10 on the num-ber of virus-resistant bacteria in each of 10 aliquots: 14 14 15 26 13 16 21 20 15 13 (a) Determine the median and the quartiles. (b) Determine the interquartile range. (c) How large would an observati

Answers

Answer:

(a)

[tex]Q_1 = 14[/tex]

[tex]Median = 15[/tex]

[tex]Q_3 = 20[/tex]

(b) [tex]IQR = 6[/tex]

Step-by-step explanation:

Given

[tex]14\ 14\ 15\ 26\ 13\ 16\ 21\ 20\ 15\ 13[/tex]

[tex]n = 10[/tex]

Solving (a): Median and the quartiles

Start by sorting the data

[tex]Sorted: 13\ 13\ 14\ 14\ 15\ 15\ 16\ 20\ 21\ 26[/tex]

The median position is:

[tex]Median = \frac{n + 1}{2}[/tex]

[tex]Median = \frac{10 + 1}{2} = \frac{11}{2} = 5.5th[/tex]

This implies that the median is the average of the 5th and the 6th data;

So;

[tex]Median = \frac{15+15}{2} = \frac{30}{2} = 15[/tex]

Split the dataset into two halves to get the quartiles

[tex]Lower: 13\ 13\ 14\ 14\ 15\[/tex]

[tex]Upper: 15\ 16\ 20\ 21\ 26[/tex]

The quartiles are the middle items of each half.

So:

[tex]Lower: 13\ 13\ 14\ 14\ 15\[/tex]

[tex]Q_1 = 14[/tex] ---- 14 is the middle item

[tex]Upper: 15\ 16\ 20\ 21\ 26[/tex]

[tex]Q_3 = 20[/tex] ---- 20 is the middle item

Solving (b): The interquartile range (IQR)

This is calculated as:

[tex]IQR = Q_3 - Q_1[/tex]

[tex]IQR = 20 - 14[/tex]

[tex]IQR = 6[/tex]

Solving (c): Incomplete details

Find the missing term in the following pattern.

320, 160, 80 blank space then, 20, 10

Answers

40

Step-by-step explanation:

Each number is followed by a number that is half its value, so the sequence goes like

320, 160, 80, 40, 20, 10.

320, 160, 80, 40, 20, 10

Which table represents a linear function?
Х
1
2
3
4
y
3
6
12
24
х
1
2
3
4
у
2.
5
9
14
х
1
2
3
4
у
-3
-5
-7
-9
х
1
2
3
4
у
-2
-4
-2
0

Answers

Answer:

3

Step-by-step explanation:

x 1,2,3,4

y-3,-5,-7,-9

[tex]y = - 3 - (x - 1) \times 2[/tex]

The linear function is given by y = 7x - 4

A linear function is in the form:

y = mx + b

where y, x are variables, m is the rate of change and b is the y intercept

From the table, using the points (1, 3) and (4, 24):

[tex]y-y_1=\frac{y_2-y_1}{x_2-x_1} (x-x_1)\\\\y-3=\frac{24-3}{4-1}(x-1)\\\\ y=7x-4[/tex]

The linear function is given by y = 7x - 4.

Find out more on linear function at: https://brainly.com/question/4025726

It says I need too put 20 characters in too ask the question so ignore this part

Answers

I believe it’s b your welcome

In a mixture of 240 gallons, the ratio of ethanol and gasoline is 3:1. If the ratio is to be 1:3, then find the quantity of gasoline that is to be added.

Answers

Answer:

480 gallons.

Step-by-step explanation:

Given that in a mixture of 240 gallons, the ratio of ethanol and gasoline is 3: 1, if the ratio is to be 1: 3, to find the quantity of gasoline that is to be added the following calculation must be performed:

240/4 x 3 = Ethanol

240/4 = Gasoline

180 = Ethanol

60 = Gasoline

0.25 = 180

1 = X

180 / 0.25 = X

720 = X

720 - 180 - 60 = X

480 = X

Therefore, 480 gallons of gasoline must be added if the ratio is to be 1: 3.

find a number such that when it is multiplied by 7 and 17 is subtracted from the product the result is the same as when it is multiplied by 3 and 19 added to the product .​

Answers

Answer:

9

Step-by-step explanation:

Let the number be X

From the problem we have the following equation:

7x - 17 = 3x + 19

-> 4x = 36

-> x = 9

Answer:

9

Step-by-step explanation:

that is the procedure above

what is 3/2 divided by 1/8

helppp​

Answers

Answer: 12

Step-by-step explanation:

The first step to dividing fractions is to find the reciprocal (reverse the numerator and denominator) of the second fraction. Next, multiply the two numerators. Then, multiply the two denominators. Finally, simplify the fractions if needed.

A Sociology instructor gives students points for each​ discussion-board post and points for each reply to a post. Ana wrote 6 posts and 8 replies and received 114 points. Jae wrote 5 posts and 4 replies and received 79 points. Determine how many points a discussion post is worth and how many points a reply is worth?

Answers

Answer:

5 points per post and 2 points per replies

1. You measure 24 textbooks' weights, and find they have a mean weight of 75 ounces. Assume the population standard deviation is 3.3 ounces. Based on this, construct a 90% confidence interval for the true population mean textbook weight.
2. You measure 37 backpacks' weights, and find they have a mean weight of 45 ounces. Assume the population standard deviation is 10.1 ounces. Based on this, construct a 95% confidence interval for the true population mean backpack weight.
3. You measure 30 watermelons' weights, and find they have a mean weight of 37 ounces. Assume the population standard deviation is 4.1 ounces. Based on this, what is the maximal margin of error associated with a 90% confidence interval for the true population mean watermelon weight.
4. A student was asked to find a 99% confidence interval for widget width using data from a random sample of size n = 16. Which of the following is a correct interpretation of the interval 11.8 < μ < 20.4?
A. There is a 99% chance that the mean of a sample of 16 widgets will be between 11.8 and 20.4.
B. The mean width of all widgets is between 11.8 and 20.4, 99% of the time. We know this is true because the mean of our sample is between 11.8 and 20.4.
C. With 99% confidence, the mean width of all widgets is between 11.8 and 20.4.
D. With 99% confidence, the mean width of a randomly selected widget will be between 11.8 and 20.4.
E. There is a 99% chance that the mean of the population is between 11.8 and 20.4.
5. For a confidence level of 90% with a sample size of 23, find the critical t value.

Answers

Answer:

(73.845 ; 76.155) ;

(41.633 ; 48.367) ;

1.273 ;

C. With 99% confidence, the mean width of all widgets is between 11.8 and 20.4. ;

1.717

Step-by-step explanation:

1.)

Given :

Mean, xbar = 75

Sample size, n = 24

Sample standard deviation, s = 3.3

α = 90%

Confidence interval = mean ± margin of error

Margin of Error = Tcritical * s/√n

Tcritical at 90% ; df = 24 - 1 = 23

Tcritical = 1.714

Margin of Error = 1.714 * 3.3/√24 = 1.155

Confidence interval = 75 ± 1.155

Confidence interval = (73.845 ; 76.155)

2.)

Given :

Mean, xbar = 45

Sample size, n = 37

Sample standard deviation, s = 10.1

α = 95%

Confidence interval = mean ± margin of error

Margin of Error = Tcritical * s/√n

Tcritical at 95% ; df = 37 - 1 = 36

Tcritical = 2.028

Margin of Error = 2.028 * 10.1/√37 = 3.367

Confidence interval = 45 ± 3.367

Confidence interval = (41.633 ; 48.367)

3.)

Given :

Mean, xbar = 37

Sample size, n = 30

Sample standard deviation, s = 4.1

α = 90%

Margin of Error = Tcritical * s/√n

Tcritical at 90% ; df = 30 - 1 = 29

Tcritical = 1.700

Margin of Error = 1.700 * 4.1/√30 = 1.273

5.)

Sample size, n = 23

Confidence level, = 90%

df = n - 1 ; 23 - 1 = 22

Tcritical(0.05, 22) = 1.717

What is the value of h in the figure below? In this diagram, ABAD ~ ACBD.
Д.
С
0
20
А. 80
В. 16
ОО ооо
С. 60
D. 5
Е. 8
о
Е.

Answers

Answer:

[tex]h=\sqrt{16(4)}[/tex]

[tex]h=8[/tex]

[tex]ANSWER:E) 8[/tex]

-------------------------------

~HOPE IT HELPS

~HAVE A GREAT DAY!!

Bob had 10 more cars than Paul. Paul had 15 cars.

Answers

Answer:

Bob had 25 cars

Step-by-step explanation:

10+15=25

Bob at 25 cars
explanation: paul had 15 cars bob had 10 more
15+10=25

Plz help I’ll mark you

Answers

Answer:

A 1/2

Step-by-step explanation:

Ratio of short length to hypotenuse

= cos60

= 1/2

y’all what are the answers

Answers

Answer:

Step-by-step explanation:

Solve for the following equation for x. l x/4 + 3 l < 6

Answers

Answer:

this is the answer I got! i don't know if it helps, but I hope it does

Kevin will start with the integers 1, 2, 3 and 4 each used exactly once and written in a row in any order. Then he will find the sum of the adjacent pairs of integers in each row to make a new row, until one integer is left. For example, if he starts with 3, 2, 1, 4, then he takes sums to get 5, 3, 5, followed by 8, 8, and he ends with the final sum 16. Including all of Kevin's possible starting arrangements of the integers 1, 2, 3 and 4, how many possible final sums are there?

Answers

Hello,

there are 5 differents sums:

16,18,20,22,24.

-------------------------------------------------------

Dim i As Integer, j As Integer, k As Integer, l As Integer, u As Integer, v As Integer, nb As Integer

Dim mat(4, 4) As Integer

nb = 0

For i = 1 To 4

   For j = 1 To 4

       If j <> i Then

           For k = 1 To 4

               If k <> j And k <> i Then

                   l = 10 - k - j - i

                   If l > 0 And l < 5 And l <> i And l <> j And l <> k Then

                       mat(1, 1) = i

                       mat(1, 2) = j

                       mat(1, 3) = k

                       mat(1, 4) = l

                       For u = 2 To 4

                           For v = 1 To 4 - u + 1

                               mat(u, v) = mat(u - 1, v) + mat(u - 1, v + 1)

                           Next v

                       Next u

                       'Call visu(mat())

                       nb = nb + 1

                       Print nb,

                       mat(4, 1)

                   End If

               End If

           Next k

       End If

   Next j

Next i

End

Sub visu (m() As Integer)

   Dim i As Integer, j As Integer

   For i = 1 To 4

       For j = 1 To 4 - i + 1

           Print m(i, j);

       Next j

       Print

   Next i

End Sub

3x+7>10
Solve for x.

Answers

Answer: x>1

Step-by-step explanation:

To solve for x, we want to isolate x.

3x+7>10       [subtract both sides by 7]

3x>3             [divide both sides by 3]

x>1

Therefore, we know that x>1.

Answer:

Step-by-step explanation:

3x + 7 > 10

3x > 10 - 7

3x > 3

x > 1

x ∈ ( 1, ∞ )

40 points! Need help finding.

Answers

The cordent plan of the answer is 2

Answer:

The scale factor will just be 2

Step-by-step explanation:

The length of PQ is twice as larger than the length of AB.

so from 12 to 6 or 6 to 12, we multiply 6 by 2 which equals to 12

math help plz
how to divide polynomials, how to understand and step by step with an example provided please

Answers

Answer:

hiiiiiii....!!! how r u

Lendo 15 páginas por dia, Marcos leu um livro em
9 dias.
Para ler esse mesmo livro em 3 dias, quantas páginas
ele deveria ler por dia?

Answers

Answer:

Olha a foto.

Step-by-step explanation:

2. The prices, in dollars per unit, of the three commodities X, Y and Z are x, y and z,
respectively
Person A purchases 4 units of Z and sells 3 units of X and 3 units of Y.
Person B purchases 3 units of Y and sells 2 units of X and 1 unit of Z.
Person C purchases 1 unit of X and sells 4 units of Y and 6 units of Z.
In the process, A, B and C earn $40, $50, and $130, respectively.

a) Find the prices of the commodities X, Y, and Z by solving a system of linear
equations (note that selling the units is positive earning and buying the units is
negative earning).​

Answers

Answer:

Price of X is $24.81

Price of Y is $3.66

Price of Z is $11.36

Step-by-step explanation:

for person A, we know that earns $40, then we can write the equation:

-4*z + 3*x + 3*y = $40

For person B, we know that earns $50, then:

1*z + 2*x - 3*y = $50

For person C, we know that earns $130, then:

6*z - 1*x + 4*y = $130

Then we have a system of equations:

-4*z + 3*x + 3*y = $40

1*z + 2*x - 3*y = $50

6*z - 1*x + 4*y = $130

To solve the system, we need to isolate one of the variables in one of the equations.

Let's isolate z in the second equation:

z = $50 - 2*x + 3*y

now we can replace this in the other two equations:

-4*z + 3*x + 3*y = $40

6*z - 1*x + 4*y = $130

So we get:

-4*($50 - 2*x + 3*y) + 3*x + 3*y = $40

6*($50 - 2*x + 3*y) - 1*x + 4*y = $130

Now we need to simplify both of these, so we get:

-$200 + 11x - 9y = $40

$350 - 13*x + 28*y = $130

Now again, we need to isolate one of the variables in one of the equations.

Let's isolate x in the first one:

-$200 + 11x - 9y = $40

11x - 9y = $40 + $200 = $240

11x = $240 + 9y

x = ($240 + 9y)/11

Now we can replace this in the other equation:

$350 - 13*x + 28*y = $130

$350 - 13*($240 + 9y)/11 + 28*y = $130

Now we can solve this for y.

- 13*($240 + 9y)/11 + 28*y  = $130 - $350 = -$220

-13*$240  - (13/11)*9y + 28y = - $220

y*(28 - (9*13/1) ) = -$220 + (13/11)*$240

y = ( (13/11)*$240 - $220)/(28 - (9*13/1) ) = $3.66

We know that:

x = ($240 + 9y)/11

Replacing the value of y, we get:

x = ($240 + 9*$3.66)/11  = $24.81

And the equation of z is:

z = $50 - 2*x + 3*y = $50 - 2* $24.81 + 3*$3.66 = $11.36

Then:

Price of X is $24.81

Price of Y is $3.66

Price of Z is $11.36

Wires manufactured for a certain computer system are specified to have a resistance of
between 0.11 and 0.16 ohm. The actual measured resistances of the wires produced by
Company A have a normal probability distribution, with a mean of 0.14 ohms, and a
standard deviation of 0.003 ohms. What is the probability that a randomly selected wire
from Company A’s production lot will meet the specifications?

Answers

Answer:

8d68d68fu9d6rf0d

c9yd7xpjd

puf68d6rif7

What is the average (with 0 decimal places) across all schools for the total score? Group of answer choices 1287 1215 1221 1229

Answers

Answer:

See explanation

Step-by-step explanation:

Required

The average

The data whose average is to be calculated are not given.

However, the formula to calculate the average is:

[tex]\bar x = \frac{\sum x}{n}[/tex]

Assume the data is:

[tex]1287\ 1215\ 1221\ 1229[/tex]

This means that the number of schools is 4

So:

[tex]\bar x = \frac{1287+ 1215+ 1221+ 1229}{4}[/tex]

[tex]\bar x = \frac{4952}{4}[/tex]

[tex]\bar x = 1238[/tex]

The average of the assumed data is 1238

Carin opened a money market account with a
deposit of $3,000. This account earns 2% simple
interest annually. How many years will it take for
her $3,000 deposit to earn $420 in interest, assum-
ing she does not withdraw any of the money?

Answers

Answer:

7

Step-by-step explanation:

For simple interest,

I = prt

where I = interest,

p = principal (amount deposited)

r = annual rate of interest

t = time in years

We have r = 2% = 0.02

p = $3,000

I = $420

We need to find t

I = prt

420 = 3000 * 0.02 * t

420 = 60t

t = 420/60

t = 7

Answer: 7 years

Find the value of x. What is the value of x?

Answers

Answer:

x = 16

Step-by-step explanation:

The product of the lengths theorem is a property that can be sued to describe the relationships of the sides between the tangents and secants in a circle. One of these products states the following;

The distance between the point of tangency and its intersection point with the exterior secant squared is equal to the product of the exterior secant times the interior secant.

This essentially means the following equation can be formed;

[tex](AB)^2=(DC)(CB)[/tex]

Substitute,

[tex]12^2=x*9[/tex]

Simplify,

[tex]144=9x[/tex]

Inverse operations,

[tex]\frac{144}{9}=x\\\\16=x[/tex]

Answer:

[tex]\boxed{\sf x=7}[/tex]

Step-by-step explanation:

By Targent-secant theorem...

[tex]\sf 9(x + 9) = {12}^{2} [/tex]

Use the distributive property to multiply 9 by x+9.

[tex]\sf 9x+81= {12}^{2} [/tex]

Now, let calculate 12 to the power of 2 and get 144.

[tex]\sf 9x+81=144[/tex]

Subtract 81 from both sides.

[tex]\sf 9x=63[/tex]

Divide both sides by 9.

[tex] \sf \cfrac{ 9x}{9} = \cfrac{63}{9} [/tex]

[tex]\sf x=7[/tex]

If the current through a circuit is 2 A and the resistance of a light bulb in the circuit is 10 Ohms what is tge voltage difference across the light bulb ​

Answers

Answer:

v = ir

2 times 10 = 20v

Step-by-step explanation:

i think it is the one

During a recent election, 54 of every 100 registered voters in a certain city voted. If there were 157,400 registered voters in that city, how many people voted?

Answers

84,996 of the registered voters voted. If 157,400 is 100 percent of the registered voters then you can take 54 percent of 157,400 (157,400 * 54) which equals 84,996

A cardboard box without a lid is to have a volume of 4,000 cm3. Find the dimensions that minimize the amount of cardboard used. (Let x, y, and z be the dimensions of the cardboard box.)

Answers

492sjnenkdoekkwjdjdkww
Other Questions
Does the narrator seem trustworthy as he gives his account of the events in the story The Tell-Tale Hearts explain how in two sentences The Purple Lion Beverage Company expects the following cash flows from its manufacturing plant in Palau over the next six years: Annual Cash Flows Year 1 Year 2 Year 3 Year 4 Year 5 Year 6 $250,000$37,500$180,000$300,000$750,000$725,000 The CFO of the company believes that an appropriate annual interest rate on this investment is 6.5%. What is the present value of this uneven cash flow stream, rounded to the nearest whole dollar 7^9 = ????? My daughter needs help with this really quick! Provide the EXACT C++ line of code necessary to return the heap-dynamic variable ptr to the operating system. How are cultural diffusion and cultural appreciation the same PLEASE HURRY perform the following series of rigid transformations on abc translate triangle abc by moving it 5 units to the right and 2 units upDraw the line y=-x and reflect Triangle ABC across the line Rotate ABC counterclockwise about the fact origin by 270 degress 50 POINTS! PLEASE HELP!________ is a VMware Network setting that does not put the virtual machine directly on the physical network of the computer. Host Only MAT NAT VNAT Many home barbeques are fueled with propane gas (C3H8)(C3H8). Part A What mass of carbon dioxide is produced upon the complete combustion of 27.9 LL of propane (the approximate contents of one 5-gallon tank) A fashion designer wants to know how many new dresses women buy each year. A sample of 650 women was taken to study their purchasing habits. Construct the 95% confidence interval for the mean number of dresses purchased each year if the sample mean was found to be 5.6. Assume that the population standard deviation is 1.3. Decide if the following scenario involves a permutation or combination. Then find the number of possibilities.The batting order for nine players on a team with 11 people. In an extensive study involving thousands of British children, Arden and Plomin (2006) found significantly higher variance in the intelligence scores for males than for females. Following are hypothetical data, similar to the results obtained in the study. Note that the scores are not regular IQ scores but have been standardized so that the entire sample has a mean of M 5 10 and a standard deviation of s 5 2. a. Calculate the mean and the standard deviation for the sample of n 5 8 females and for the sample of n 5 8 males. b. Based on the means and the standard deviations, describe the differences in intelligence scores for males and females. The penalties for a first-time DUI charge include imprisonment for __________.A. 180 daysB. up to six months for a BAL of .08 to .15C. up to one year if there is a minor in the vehicleD. B and C what is the value of x? no links. Which formula can be used to describe the sequence?O f(x + 1) = f(x)O f(x + 1) = - f(x)O f(x) = f(x + 1)O f(x) = - 3 f(x + 1) The expression 2x and x have the same value for only two values of x. What are these values? 21. In each of the following situations, attribute theFundamental Right guaranteed:a. A child of an HIV positive parent seeks admission in asecondary school.b. In a remote village a poor woman is forbidden to takewater from a particular well, though all the other wellsare dryc. Atul's father has been transferred to a beautiful hillstation after living in the plains for a long time.d. The poor labourer has sent his minor son to work inthe fireworks factorye. John goes to the church for the morning mass onevery Sunday, while his friend Ali attends eveningprayers in the mosque on Fridays and their colleagueArjun offers flowers at the feet of goddess Saraswati. How does alphabet knowledge connect to childrens later success with decoding? Lorenzos test scores are shown below.Lorenzos Test ScoresA dot plot titled Lorenzo's Test Scores. A number line going from 80 to 90. 0 dots are above 80, 81, and 82, there is 1 dot above 83, 2 above 84, 2 above 85, 2 above 86, 2 above 87, 0 above 88 and 89, and 1 above 90.Anikas test scores are shown below.Anikas Test ScoresA dot plot titled Anika's Test Scores. A number line going from 80 to 90. There are 0 dots above 80, 81, 82, and 83, 3 dots above 84, 2 above 85, 2 above 86, 3 above 87, and 0 dots above 88, 89, and 90.Which statement compares the shape of the two dot plots?There is a gap in both plots.There is a gap in Anikas scores, but not in Lorenzos scores.The data is widely spread across both plots.The data is more widely spread for Lorenzos scores than for Anikas. Could you pleaseeee help me i need some help i need it before tomorrow at 4:00pm Please help me with this one